[別館]球面倶楽部零八式markIISR

東大入試数学中心。解説なので解答としては不十分。出題年度で並ぶようにしている。大人の解法やうまい解法は極めて主観的に決めている。

2024年(令和6年)防衛医科大学校医学科-数学[5](数字)

2023.10.29記(2024.02.20修正)

[5] \angle\mbox{A}=\dfrac{5}{9}\pi\angle\mbox{B}=\dfrac{5}{18}\pi\angle\mbox{C}=\dfrac{1}{6}\pi\triangle\mbox{ABC} において,辺 \mbox{AC} 上に \angle\mbox{ABD}=\dfrac{1}{6}\pi なる点 \mbox{D} をとる.また,点 \mbox{A} から辺 \mbox{BC} に垂線を下ろし,辺 \mbox{BC} との交点を \mbox{H} とし,直線 \mbox{AH} と 直線 \mbox{BD} の交点を \mbox{E}とする.\dfrac{\tan\dfrac{5}{18}\pi\,\tan\dfrac{7}{18}\pi}{\tan\dfrac{1}{3}\pi \,\tan\dfrac{4}{9}\pi}=\dfrac{\fbox{ 11 }}{\fbox{ 12 }} となるため,\angle\mbox{CEH}=\dfrac{\fbox{ 13 }}{\fbox{ 14 }}\pi である.(分数はそれ以上約分できない形で解答すること.)

2023.10.29記(2024.02.20修正)
(1) \tan50^{\circ}\tan60^{\circ}\tan70^{\circ}=\tan80^{\circ}という有名性質から\dfrac{1}{3}となる,
という訳にもいかないので…

図をそこそこ正確に書くと,(2)の答が \dfrac{4}{9}\pi になるはずだと思って計算すると (1) は \dfrac{1}{3} でなければならない,という手もある.

[解答]
一般に \triangle\mbox{ABC} の内角において, \tan(A+B)=\dfrac{\tan A+\tan B}{1-\tan A\tan B}=\tan(\pi-C)=-\tan C から \tan A\tan B\tan C=\tan A+\tan B + \tan C が成立する.

よって
\tan\dfrac{5}{18}\pi\,\tan\dfrac{\pi}{3}\pi\,\tan\dfrac{7}{18}\pi
=\tan 50^{\circ}+\tan 60^{\circ}+\tan 70^{\circ}
=\dfrac{\sin 50^{\circ}\cos 60^{\circ}+\sin 60^{\circ}\cos 50^{\circ}}{\cos 50^{\circ}\cos 60^{\circ}}+\dfrac{\sin 70^{\circ}}{\cos 70^{\circ}}
=\dfrac{\sin 110^{\circ}}{\sin 40^{\circ}(1/2)}+\dfrac{\cos 20^{\circ}}{\sin 20^{\circ}}
=\dfrac{\cos 20^{\circ}}{\sin 20^{\circ}\cos 20^{\circ}}+\dfrac{\cos 20^{\circ}}{\sin 20^{\circ}}
=\dfrac{1+\cos 20^{\circ}}{\sin 20^{\circ}}=\dfrac{2\cos^2 10^{\circ}}{2\sin10^{\circ}\cos10^{\circ}}=\dfrac{\cos 10^{\circ}}{\sin10^{\circ}}=\tan80^{\circ}
となる.以上から
\tan50^{\circ}\tan60^{\circ}\tan70^{\circ}=\tan80^{\circ}
つまり
\tan\dfrac{5}{18}\pi\,\tan\dfrac{\pi}{3}\pi\,\tan\dfrac{7}{18}\pi=\tan\dfrac{4}{9}\pi
が成立する.

よって \dfrac{\tan\dfrac{5}{18}\pi\,\tan\dfrac{7}{18}\pi}{\tan\dfrac{\pi}{3}\,\tan\dfrac{4}{9}\pi}=\tan^2\dfrac{\pi}{3}=\dfrac{1}{3} となる.

このとき
3\tan\dfrac{5}{18}\pi\,\tan\dfrac{7}{18}\pi=\tan\dfrac{\pi}{3}\cdot\tan\dfrac{4}{9}\pi
だから,
\tan\dfrac{\pi}{3}\,\tan\dfrac{5}{18}\pi\,\tan\dfrac{7}{18}\pi=\tan\dfrac{4}{9}\pi
となる.
\angle\mbox{ABH}=\dfrac{5}{18}\piだから\tan\dfrac{5}{18}\pi=\dfrac{\mbox{AH}}{\mbox{BH}}
\angle\mbox{BEH}=\dfrac{7}{18}\piだから\tan\dfrac{7}{18}\pi=\dfrac{\mbox{BH}}{\mbox{EH}}
\angle\mbox{CAH}=\dfrac{\pi}{3}だから\tan\dfrac{\pi}{3}=\dfrac{\mbox{CH}}{\mbox{AH}}
により\dfrac{\mbox{CH}}{\mbox{EH}}=\tan\dfrac{4}{9}\piであるから,\angle\mbox{CEH}=\dfrac{4}{9}\pi

よって \dfrac{\fbox{ 11 }}{\fbox{ 12 }}=\dfrac{1}{3}\dfrac{\fbox{ 13 }}{\fbox{ 14 }}=\dfrac{4}{9} である.

(1) は普通は次のようにやる.

\dfrac{\tan\dfrac{5}{18}\pi\,\tan\dfrac{7}{18}\pi}{\tan\dfrac{\pi}{3}\,\tan\dfrac{4}{9}\pi}=\dfrac{\tan 50^{\circ}\,\tan 70^{\circ}}{\sqrt{3}\,\tan 80^{\circ}}=\dfrac{1}{\sqrt{3}}\tan 50^{\circ}\,\tan 70^{\circ}\,\tan 10^{\circ}
=\dfrac{1}{\sqrt{3}}\cdot\dfrac{\sin 50^{\circ}\,\sin 70^{\circ}\,\sin 10^{\circ}}{\cos 50^{\circ}\,\cos 70^{\circ}\,\sin 80^{\circ}}
=\dfrac{1}{\sqrt{3}}\cdot\dfrac{\cos 40^{\circ}\,\cos 20^{\circ}\,\sin 10^{\circ}}{\sin 40^{\circ}\,\sin 20^{\circ}\,\sin 80^{\circ}}
=\dfrac{1}{\sqrt{3}}\cdot\dfrac{\cos 40^{\circ}\,\cos 20^{\circ}\,\sin 10^{\circ}}{8\sin 20^{\circ}\cos20^{\circ}\,\sin 10^{\circ}\cos10^{\circ}\,\sin 40^{\circ}\cos40^{\circ}}
=\dfrac{1}{\sqrt{3}}\cdot\dfrac{1}{8\sin 20^{\circ}\,\cos10^{\circ}\,\sin 40^{\circ}}
=\dfrac{1}{\sqrt{3}}\cdot\dfrac{1}{8\sin 20^{\circ}\,\sin40^{\circ}\,\sin 80^{\circ}}
=\dfrac{1}{\sqrt{3}}\cdot\dfrac{1}{4(\cos 20^{\circ}-\cos 60^{\circ})\,\sin80^{\circ}}
=\dfrac{1}{2\sqrt{3}}\cdot\dfrac{1}{\sin 60^{\circ}+\sin 100^{\circ}-\sin80^{\circ}}
=\dfrac{1}{2\sqrt{3}}\cdot\dfrac{1}{\sin 60^{\circ}}=\dfrac{1}{3}
ちょっと長いね.

\dfrac{\pi}{18}=\theta\tan\theta=\alpha とおくと,
\dfrac{\tan\dfrac{5}{18}\pi\,\tan\dfrac{7}{18}\pi}{\tan\dfrac{\pi}{3}\,\tan\dfrac{4}{9}\pi}=\dfrac{\tan\left(\dfrac{\pi}{3}-\theta\right)\tan\left(\dfrac{\pi}{3}+\theta\right)}{\tan\dfrac{\pi}{3}\cdot\dfrac{1}{\tan\theta}}
を加法定理で展開すると,\alpha=\tan\theta の式で表現できることがわかる.これが最終的に有理数になるはずと思えば,\tan3\theta=\dfrac{1}{\sqrt{3}} をうまく使うことになると考えて,
\tanの3倍角の公式により,
\dfrac{3\alpha-\alpha^3}{1-3\alpha^2}=\tan3\theta=\tan\dfrac{\pi}{6}=\dfrac{1}{\sqrt{3}}
となることを用いるはずであると考えよう.すると,

\dfrac{\tan\dfrac{5}{18}\pi\,\tan\dfrac{7}{18}\pi}{\tan\dfrac{\pi}{3}\,\tan\dfrac{4}{9}\pi}=\dfrac{\tan\left(\dfrac{\pi}{3}-\theta\right)\tan\left(\dfrac{\pi}{3}+\theta\right)}{\tan\dfrac{\pi}{3}\cdot\dfrac{1}{\tan\theta}}
=\dfrac{1}{\sqrt{3}}\cdot\dfrac{\sqrt{3}-\alpha}{1+\sqrt{3}\alpha}\cdot\dfrac{\sqrt{3}+\alpha}{1-\sqrt{3}\alpha}\cdot\alpha
=\dfrac{1}{\sqrt{3}}\cdot\dfrac{3\alpha-\alpha^3}{1-3\alpha^2}
=\dfrac{1}{\sqrt{3}}\cdot\tan 3\theta
=\dfrac{1}{\sqrt{3}}\cdot\dfrac{1}{\sqrt{3}}=\dfrac{1}{3}
となる.